Proving Differentiability of a Piece-wise Function

Click For Summary
The discussion centers on the differentiability of the piecewise function f(x), defined as 0 for irrational x and x for rational x, specifically at x=0. The derivative is evaluated using the limit definition, leading to two different results: 1 for rational inputs and 0 for irrational inputs. Since these two limits do not match, it concludes that f'(0) does not exist. The reasoning emphasizes the presence of both rational and irrational numbers arbitrarily close to zero, akin to evaluating left-hand and right-hand limits. Thus, the function is confirmed to be non-differentiable at that point.
UziStuNNa
Messages
9
Reaction score
0
1. Suppose f(x)=0 if x is irrational, and f(x)=x if x is rational. Is f differentiable at x=0?



2. the derivative= lim[h->0] [f(a+h)-f(a)]/h



3. I don't really know how to start, but I do know that between any two real numbers, there exists a rational and irrational number. So I'm guessing that has something to do with solving for the answer. f'(0)= lim[h->0] f(h)/h
 
Physics news on Phys.org
Ok, so there are rational numbers as close as you want to 0. And irrational numbers. Pick h to be rational. What's f(h)/h? Same question for irrational.
 
For rational,
f'(0)= lim[h->0] f(h)/h= h/h=1

For irrational,
f'(0)= lim[h->0] f(h)/h= 0/h=0

Therefore, the two limits do not equal each other, meaning that f'(0) does not exist?
Was it that easy?
 
UziStuNNa said:
For rational,
f'(0)= lim[h->0] f(h)/h= h/h=1

For irrational,
f'(0)= lim[h->0] f(h)/h= 0/h=0

Therefore, the two limits do not equal each other, meaning that f'(0) does not exist?
Was it that easy?

Yes. If you are clear why you can pick h values arbitrarily close to zero that are both rational and irrational. And I think you are.
 
So its like doing the left-hand and right-hand limits, except we are using the piece-wise function to our advantage knowing that x can be rational and irrational, and since there are an infinite number of rational and irrational numbers approaching zero, those are our 'left' and 'right'.

Thanks a lot.
 
UziStuNNa said:
So its like doing the left-hand and right-hand limits, except we are using the piece-wise function to our advantage knowing that x can be rational and irrational, and since there are an infinite number of rational and irrational numbers approaching zero, those are our 'left' and 'right'.

Thanks a lot.

Exactly. Very welcome.
 
Question: A clock's minute hand has length 4 and its hour hand has length 3. What is the distance between the tips at the moment when it is increasing most rapidly?(Putnam Exam Question) Answer: Making assumption that both the hands moves at constant angular velocities, the answer is ## \sqrt{7} .## But don't you think this assumption is somewhat doubtful and wrong?

Similar threads

  • · Replies 5 ·
Replies
5
Views
2K
  • · Replies 8 ·
Replies
8
Views
1K
Replies
26
Views
2K
  • · Replies 6 ·
Replies
6
Views
2K
Replies
6
Views
2K
Replies
8
Views
2K
  • · Replies 8 ·
Replies
8
Views
2K
  • · Replies 11 ·
Replies
11
Views
2K
Replies
1
Views
2K
  • · Replies 11 ·
Replies
11
Views
2K